This site is currently being migrated at a new site. Please read the information below.

LaTeX

Unicode

Showing posts with label real analysis. Show all posts
Showing posts with label real analysis. Show all posts

Monday, December 5, 2016

A double Putnam 2016 series

Evaluate the series:

$$\sum_{k=1}^{\infty}\frac{(-1)^{k-1}}{k}\sum_{n=0}^{\infty}\frac{1}{k2^n+1}$$

(Putnam 2016)
Solution

Tuesday, November 15, 2016

Not Lebesgue integrable function

Let $x \in \mathbb{R}$. Given the series:

\begin{equation} \sum_{n=2}^{\infty} \frac{\sin nx}{\ln n} \end{equation}
  1. Prove that $(1)$ converges forall $x \in \mathbb{R}$.
  2. Prove that $(1)$ is not a Fourier series of a Lebesgue integrable function.
Solution

Thursday, November 3, 2016

$\mathbb{R}^2 \rightarrow \mathbb{R}$

Prove that there does not exist an $1-1$ and continuous mapping from $\mathbb{R}^2$ to $\mathbb{R}$.

Solution

Wednesday, October 26, 2016

Existence of constant

Let $f:[0, 1] \rightarrow \mathbb{R}$ be a continuous function such that $f(0)=0$ and

$$\int_0^1 f(x) \, {\rm d}x = \int_0^1 x f(x) \, {\rm d}x \tag{1}$$

Prove that there exists a $c \in (0, 1)$ such that

$$\int_0^c x f(x) \, {\rm d}x = \frac{c}{2} \int_0^c f(x) \, {\rm d}x$$

Solution

Friday, October 14, 2016

Convergence and dyadic numbers

A real number $x$ is said to be dyadic rational provided there is an integer $k$ and a non negative integer $n$ for which $\displaystyle x=\frac{k}{2^n}$ . For each $x \in [0, 1]$ and each $n \in \mathbb{N}$ set:

$$f_n(x) = \left\{\begin{matrix} 1 &, & x =\dfrac{k}{2^n} , \; k \in \mathbb{N} \\ 0& , & \text{otherwise} \end{matrix}\right.$$
  1. Prove that the dyadic numbers are dense in $\mathbb{R}$.
  2. Let $f:[0, 1] \rightarrow \mathbb{R}$ be the function to which the sequence $\{f_n\}_{n \in \mathbb{N}}$ converges pointwise. Prove that $\bigintsss_0^1 f(x) \, {\rm d}x$ does not exist.
  3. Show that the convergence $f_n \rightarrow f$ is not uniform.
Solution

Thursday, October 13, 2016

An evaluation of integral with unknown $f$

Let $f:\mathbb{R} \rightarrow \mathbb{R}$ be a differentiable function in $[0, 1]$, strictly monotonic and $f(0)=1$ . If forall $x \in \mathbb{R}$ holds $f\left( f(x) \right)=x$ then evaluate the integral

$$\mathcal{J} = \int_0^1 \left(x - f(x) \right)^{2016} \, {\rm d}x$$

Solution

Wednesday, October 12, 2016

Integral and inequality

Let $f:\mathbb{R} \rightarrow \mathbb{R}$ be a positive real valued and continuous function such that it is periodic of period $T=1$. Prove that:

$$\int_0^1 \frac{f(x)}{f \left(x + \frac{1}{2} \right)}\, {\rm d}x  \geq 1$$

Solution

Monday, September 19, 2016

Probably a beauty and a beast

Here is something that arose while playing around with a class of particular integrals.

Let $\displaystyle a_n= \int_0^1 \left( e^x - 1 - x - \frac{x^2}{2} - \cdots - \frac{x^n}{n!} \right) \, {\rm d}x$. Evaluate the series

$$\mathcal{S}=\sum_{n=0}^{\infty} a_n$$

Solution

Thursday, September 15, 2016

On functions

Here are some examples on functions with "strange" properties.

Give an example of a function that:


  1. is only continuous at a single point.
  2. is continuous at isolated points.
Does there exist a function that:
  1. is continuous only on the rationals?
  2. is continuous only on the irrationals?
Explain your answer giving a brief explanation. 

Solution

Tuesday, September 13, 2016

Double alternating sum

Let $\displaystyle c_n= \sum_{k=1}^{\infty} \frac{(-1)^{k-1}}{k+n}$. Evaluate the sum:

$$\mathcal{S}=\sum_{n=1}^{\infty} \frac{c_n}{n}$$

Solution

Monday, September 12, 2016

A beautiful double sum

Prove that:

$$ \sum_{n=1}^{\infty}\left [ \frac{1}{n} \sum_{m=0}^{\infty} \frac{1}{(2m+1)^{2n}} - \log \left ( 1 + \frac{1}{n} \right ) \right ]$$

Solution

Tuesday, August 30, 2016

On a known inequality

Let $x$ be a real number and let $n \in \mathbb{N}$. Prove the inequality:

$$\left | \sum_{k=1}^{n} \frac{\sin kx}{k} \right |\leq 2 \sqrt{\pi}$$

Solution

Wednesday, August 10, 2016

Zero function

The following is an exercise proposed by one of our readers.

Let $f:\mathbb{R} \rightarrow \mathbb{R}$ be a continuous function such that $f\left( \frac{m}{2^n} \right)=0 \; \text{forall} \; m \in \mathbb{Z} \; \text{and} \; n \in \mathbb{N}$. Prove that $f=0$ forall $x \in \mathbb{R}$.

Solution

Saturday, July 30, 2016

An equality of two finite sums

Let $n \in \mathbb{N}$. Prove that:

$$\sum_{k=1}^{n}\frac{1}{4k^{2}-2k}=\sum_{k=n+1}^{2n}\frac{1}{k}$$

Solution

A limit with floor function

Let $\left \lfloor \cdot \right \rfloor$ denote the floor function. Prove that:

$$\lim_{x\rightarrow 0} \; x \left \lfloor \frac{1}{x} \right \rfloor=1$$

Solution

Sunday, June 12, 2016

Example of a function

Give an example of a continuous function $f:[1, +\infty) \rightarrow \mathbb{R}$ such that $f(x)>0$ forall $x \in [1, +\infty)$ and $\bigintsss_{1}^{\infty} f(x) \, {\rm d}x$ converges while $\bigintsss_{1}^{\infty} f^2(x) \, {\rm d}x$ diverges.

Solution

Wednesday, June 1, 2016

Constant as periodic

Let $f:\mathbb{R} \rightarrow \mathbb{R}$ be a periodic function such that $\lim \limits_{x \rightarrow +\infty} f(x) =\ell \in \mathbb{R}$. Prove that $f$ is constant.

Solution

Sunday, April 3, 2016

$\mathbb{K}_n$ series

The $\mathbb{K}_n$ series ( or Kempner series) are obtained by removing all terms containing a single digit $d$ from the harmonic series. For example:

$$\mathbb{K}_1= \frac{1}{2}+ \frac{1}{3}+ \frac{1}{4}+ \frac{1}{5}+ \frac{1}{6}+ \frac{1}{7}+ \frac{1}{8}+ \frac{1}{9}+ \frac{1}{20}+ \frac{1}{22}+ \frac{1}{23}+\cdots$$

All $\mathbb{K}$ ten series converge as strangly as it sounds. In this topic we shall prove the convergence of $\mathbb{K}_1$.

Solution

Wednesday, March 16, 2016

Constant function as periodic

Prove that every continuous and periodic function that has every $\frac{1}{n}$ as a period is constant.

Solution

Wednesday, March 9, 2016

Absolute convergence of a series

Let $y_n , \; n \in \mathbb{N}$ be a sequence of real numbers such that for all real valued sequences $x_n , \; n \in \mathbb{N}$ such that $\lim x_n =0$ the series $\sum \limits_{n=1}^{\infty} x_n y_n $ converges. Does it necessarily follow that the series $\sum \limits_{n=1}^{\infty} \left| y_n \right|$ converges?

Solution